0 Daumen
3,5k Aufrufe

Ich komme mit einer Aufgabe nicht weiter.

Die Aufgabe lautet:

Bestimmen Sie den größten und den kleinsten Funktionswert, den die Funktion annehmen kann.

f ( x, y, z) = 3x + y^2 + z^3   auf Definitonsbereich = { (x,y,z) ∈ℝ^3 |  x ≥ 0,   y ≥ 0,   z ≥ 0,   x+y+z = 3 }


Zuerst hatte ich gedacht, dass ich mit ausprobieren (Wertetabelle) einen Erfolg haben könnte - leider nicht. Die Aufgabe verstehe ich zwar soweit, aber ich finde absolut keinen Ansatz, wie ich herangehen kann.

Könntet ihr mir bitte helfen? :)

Avatar von
f ( x, y, z) = 3x + y2 + z3   auf
Definitonsbereich = { (x,y,z) ∈ℝ3 |  x ≥ 0,   y ≥ 0,   z ≥ 0,  
x+y+z = 3 }

Vorschlag :

Der größte Funktionswert dürfte bei
x = 0
y = 0
z = 3 sein.
Summe : 27

Der kleinste Funktionswert dürfte bei
x = 1
y = 1
z = 1 sein.
Summe ; 5
Eher nicht.$$f\left(\frac34,\frac98,\frac98\right)<f(1,1,1).$$
Die 1.Antwort von Gast
x = 0.5
y = 1.5
z = 1
ist richtig

Die Vorgehensweise ist
Eine Unbekannte durch die beiden anderen ersetzen
x = 3 - y - z
Dann bekommt man eine Funktion mit 2 Unbekannten.
Diese dann partiell ableiten und die Extrema finden.

Bild Mathematik


Lieben Dank euch allen! Ich habe es nun viel besser verstanden, habe jedoch noch kleine Verständigungsprobleme.

a) Wenn beim Schritt "Eine Unbekannte durch die beiden anderen ersetzen
x = 3 - y - z" herauskommt, wieso wird dann die 3 nicht auch Minus? Bzw. wieso bleibt diese positiv?

b) Wieso wird es dann bei der Ableitung von y' -> -3 und +2y? Was übersehe ich gerade ? :)

b) Ich denke, dass ich noch einen Antwortsatz schreiben müsste. Der größte und kleinste Funktionswert wäre also demnäch (wenn ich es richtig verstanden habe) y = 3/2 und x = 0,5?


Vielen Dank noch mal! :)

1. wat heißt eatupyourgreens ?

a) Wenn beim Schritt "Eine Unbekannte durch die beiden
anderen ersetzen  x = 3 - y - z" herauskommt, wieso wird dann
die 3 nicht auch Minus? Bzw. wieso bleibt diese positiv?

x + y + z = 3  | - y
x + z = 3 - y  | - z
x = 3 - y - z

Das ist aber eigentlich eine einfache Umformung. Grins.

b) Wieso wird es dann bei der Ableitung von y' -> -3 und +2y?
Was übersehe ich gerade ? :)

f ( y,z ) = 3 * ( 3 - y - z ) + y^2 + z^3
f ( y,z ) = 9  -3* y - 3*z  + y^2 + z^3
Ableitung nach y : z wird als Konstante angesehen und entfällt
f´ y = -3 + 2 * y
Ableitung nach z : y wird als Konstante angesehen und entfällt
f ´ z  = -3 + 3 * z^2

c) Ich denke, dass ich noch einen Antwortsatz schreiben müsste.
Der größte und kleinste Funktionswert wäre also demnäch
(wenn ich es richtig verstanden habe) y = 3/2 und x = 0,5?

Nein.
Der größte Funktionswert ist bei
x = 0
y = 0
z = 3 und beträgt 27 : f ( 0,0,3 )

Der kleinste Funktionswert ist bei
x = 0.5
y = 1.5
z = 1 und  beträgt 4.75 : f ( 0.5, 1.5, 1 )

@georg: Du musst die Funktion noch auf Randextrema untersuchen, sonst findet du nicht alle Extrema (das hätte dir auch selbst auffallen müssen, denn du hast nur eine Extremstelle gefunden (bei der man noch überprüfen muss, ob es Maximum, Minimum oder Sattelpunkt ist), es ist aber nach dem größten  und kleinsten Funktionswert gefragt.

Schönen Dank für den Hinweis.

Bisher war ich davon ausgegangen
z = 3 ; z^3 = höchster Funktionswert

Da nur 1 anderes Extremum gefunden wurde und der
Funktionswert 4.75 den bisher kleinsten gefunden Funktionswert
darstellt habe ich das Minimum angenommen.

Mit den Randextrema wird es bei mir schwierig.
Was wären die Ränder die für x,y,z einzusetzen wären ?

Für die Funktion, bei der du schon \(x\) substituiert hast, in der also noch nur \(y\) und \(z\) vorkommen, gibt es die Einschränkungen \(y\geq 0, z\geq 0\) und \(y+z\leq 3\). Wenn du dir diese Menge mal aufzeichnest, siehst du, welche Punkte auf dem Rand dieser Menge liegen: \(\{(y,0)\in\mathbb R^2\mid 0\leq y\leq 3\}\cup \{(0,z)\in\mathbb R^2\mid 0\leq z\leq 3\}\cup \{(y,3-y)\in\mathbb R^2\mid 0\leq y\leq 3\}\).


D.h. du setzt einmal \(y=0\) ein: \(f(0,z)=3(3-z)+z^3\). Von dieser Funktion \(z\mapsto 3(3-z)+z^3\) wieder die Extrema auf dem Intervall \([0,3]\) bestimmen (dabei die gesonderte Betrachtung der Randpunkte dieses Intervalls nicht vergessen): Man erhält dann ein Maximum bei \(z=3\) und ein Minimum bei \(z=1\).

Genauso dann nochmal für \(z=0\) und für \(z=y-3\).

Dann noch die entsprechenden Funktionswerte zu all diesen Randmaxima/-minima berechnen und dann kann man entscheiden, was das globale Maximum/Minimum der Ausgangsfunktion ist.

2 Antworten

+1 Daumen
 
Beste Antwort


ersetze \(x\) durch \(x=3-y-z\). Die Funktion lautet dann $$f(y,z)=9-3y-3z+y^2+z^3=\left(y-\tfrac32\right)^2+\left(z^3-3z+\tfrac{27}4\right)$$und nimmt ihr Minimum offenbar bei \(y=\tfrac32\) und \(z=1\) und damit \(x=\frac12\) an.
Avatar von
0 Daumen

Versuch mal mit Lagrange-Funktion.

L(x,y,z,l) = 3x + y2 + z3  + l *(x+y+z - 3)

Avatar von 288 k 🚀

Ein anderes Problem?

Stell deine Frage

Willkommen bei der Mathelounge! Stell deine Frage einfach und kostenlos

x
Made by a lovely community